Đến nội dung

Hình ảnh

Topic nhận đề bất đẳng thức, cực trị hoặc toán rời rạc


  • Chủ đề bị khóa Chủ đề bị khóa
Chủ đề này có 16 trả lời

#1
E. Galois

E. Galois

    Chú lùn thứ 8

  • Quản lý Toán Phổ thông
  • 3861 Bài viết

Topic này dùng để BTC nhận đề thi từ các toán thủ thi đấu về bất đẳng thức, cực trị hoặc toán rời rạc

 

I- Bạn cần biết:

1) Điều lệ giải đấu

2) Lịch thi đấu

3) Đăng kí thi đấu

 

II - Yêu cầu về đề bài
1. Hình thức:

- Đề bài phải có đáp án kèm theo.

- Đề bài và đáp án được gõ $\LaTeX$ rõ ràng

2. Nôi dung

* Đối với MSS

- Mỗi bộ đề bao gồm 1 câu của THCS. Kiến thức dùng để giải bài không vượt quá kiến thức thi tuyển sinh vào 10. 

- Đề bài không được ở dạng thách đố, cách giải ngặt ngèo thông qua những bổ đề quá khó, không copy nguyên văn từ đề thi tuyển sinh vào 10, Olympic hoặc HSG cấp tỉnh trở lên.

- Toán thủ không nên copy đề bài từ một topic nào đó của VMF, không được post lại đề đã nộp ra topic mới dù cho đề có được chọn hay không.

 

III - Mẫu đăng kí và nộp đề

1. Họ và tên thật:

2. Đang học lớp ?, trường ?, huyện ?, tỉnh ?

3. Đề 

4. Đáp án

 

IV - Chú ý

1) Bạn sẽ thấy ở trên khung trả lời của bạn có dòng sau Bài viết này phải qua kiểm duyệt của quản trị viên mới được đăng lên diễn đàn.

Điều này có nghĩa là các toán thủ khi nộp đề, cứ yên tâm rằng, sau khi đánh máy và ấn nút GỬI BÀI là đề đã được lưu, BTC đã nhận được đề của bạn, có điều bạn không nhìn thấy được mà thôi. Bạn nên mừng vì điều này, như thế các toán thủ khác không thể biết trước đề của bạn được.

 

2) Bạn cũng nên sử dụng chức năng xem trước của diễn đàn để sửa các lỗi $\LaTeX$ trước khi gửi bài, vì gửi rồi sẽ không xem và sửa lại được nữa. 

 

3) Nếu đề bài của bạn không được chấp nhận, BTC sẽ làm hiện nó và nói rõ lý do vì sao, khi đó, bạn phải nộp đề khác. 

Nếu đề bài của bạn được chấp nhận, bạn sẽ thấy tên mình trong danh sách thi đấu tại đây sau mỗi thứ 7 hàng tuần.

 

4) Mỗi tuần, BTC chỉ cho phép toán thủ đăng kí 1 nộp đề cho 1 chủ đề nên bạn đừng ngạc nhiên khi thấy có lúc topic này bị khóa

 

 


1) Xem cách đăng bài tại đây
2) Học gõ công thức toán tại: http://diendantoanho...oạn-thảo-latex/
3) Xin đừng đặt tiêu đề gây nhiễu: "Một bài hay", "... đây", "giúp tớ với", "cần gấp", ...
4) Ghé thăm tôi tại 
http://Chúlùnthứ8.vn

5) Xin đừng hỏi bài hay nhờ tôi giải toán. Tôi cực gà.


#2
letankhang

letankhang

    $\sqrt{MF}'s$ $member$

  • Thành viên
  • 1079 Bài viết

1. Họ và tên thật: Lê Tấn Khang

2. Đang học lớp 8/3, trường Nguyễn Bỉnh Khiêm, thành phố Biên Hòa, tỉnh Đông Nai

3. Đề  EM đã đăng kí thành công và xin nộp thêm đề

Cho $a;b;c;d>0$. Chứng minh rằng :

$\frac{a^{2}}{b^{5}}+\frac{b^{2}}{c^{5}}+\frac{c^{2}}{d^{5}}+\frac{d^{2}}{a^{5}}\geq \frac{1}{a^{3}}+\frac{1}{b^{3}}+\frac{1}{c^{3}}+\frac{1}{d^{3}}$

4. Đáp án :

Áp dụng BĐT Cauchy cho 5 số dương : $\frac{a^{2}}{b^{5}};\frac{a^{2}}{b^{5}};\frac{a^{2}}{b^{5}};\frac{1}{a^{3}};\frac{1}{a^{3}}$

$\Rightarrow 3\frac{a^{2}}{b^{5}}+2\frac{1}{a^{3}}\geq 5\sqrt[5]{\frac{a^{6}}{b^{15}a^{6}}}=5\frac{1}{b^{3}}$

Tương tự : $\Rightarrow 3\frac{b^{2}}{c^{5}}+2\frac{1}{b^{3}}\geq 5\frac{1}{c^{3}};3\frac{c^{2}}{d^{5}}+2\frac{1}{c^{3}}\geq 5\frac{1}{d^{3}};3\frac{d^{2}}{a^{5}}+2\frac{1}{d^{3}}\geq 5\frac{1}{a^{3}}$

Cộng tất cả vế theo vế, sau khi rút gọn ta sẽ được :

$\frac{a^{2}}{b^{5}}+\frac{b^{2}}{c^{5}}+\frac{c^{2}}{d^{5}}+\frac{d^{2}}{a^{5}}\geq \frac{1}{a^{3}}+\frac{1}{b^{3}}+\frac{1}{c^{3}}+\frac{1}{d^{3}}$

Dấu $=$ xảy ra $\Leftrightarrow a^{5}=b^{5};b^{5}=c^{5};c^{5}=d^{5};d^{5}=a^{5}\Leftrightarrow a=b=c=d$

 

 


        :oto:   :nav:  :wub:  $\mathfrak Lê $ $\mathfrak Tấn $ $\mathfrak Khang $ $\mathfrak tự$ $\mathfrak hào $ $\mathfrak là $ $\mathfrak thành $ $\mathfrak viên $ $\mathfrak VMF $  :wub:   :nav:  :oto:            

  $\textbf{Khi đọc một quyển sách; tôi chỉ ráng tìm cái hay của nó chứ không phải cái dở của nó.}$

 

 


#3
No VND

No VND

    Lính mới

  • Thành viên
  • 2 Bài viết

1,Họ và tên: Nguyễn Tiến Sang

2,Lớp 8A, Trường THCS Cẩm Nhượng, Huyện Cẩm Xuyên, Tỉnh Hà Tĩnh

3,Đề : Cho a,b,c dương. Chứng minh rằng :

$\frac{1}{a}+\frac{1}{b}+\frac{1}{c}\geq 2(\frac{b\sqrt{a}}{a^{2}+b^{3}}+\frac{c\sqrt{b}}{b^{2}+c^{3}}+\frac{a\sqrt{c}}{c^{2}+a^{3}})$

4, Đáp án :

Ta có: $\frac{1}{a}+\frac{1}{b}=\frac{a}{a^{2}}+\frac{b^{2}}{b^{3}}\geq \frac{2b\sqrt{a}}{\sqrt{a^{2}b^{3}}}\geq 4\frac{b\sqrt{a}}{a^{2}+b^{3}}$  ( BĐT Cô-Sy)

Dấu bằng xảy ra khi và chỉ khi a=b=1

Tương tự : $\frac{1}{b}+\frac{1}{c}\geq 4\frac{c\sqrt{b}}{b^{2}+c^{3}}$

$\frac{1}{c}+\frac{1}{a}\geq 4\frac{a\sqrt{c}}{c^{2}+a^{3}}$

Cộng vế với vế của các bất đẳng thức trên suy ra điều phải chứng minh.

Dấu bằng xảy ra khi và chỉ khi a=b=c=1



#4
nguyentrungphuc26041999

nguyentrungphuc26041999

    Sĩ quan

  • Thành viên
  • 406 Bài viết

Họ và tên : Nguyễn Trung Phúc.

Đang học lớp 9A , Trường THCS Đặng Thai Mai, thành phố Vinh ,Tỉnh Nghệ An

Đề

Cho $a,b,c$ là các số thực dương, thoả mãn $a+b+c=3$

Chứng minh rằng

$\sum \frac{1}{4a^{2}+b^{2}+c^{2}}\leq \frac{1}{2}$

Lời giải

Sử dụng bất đẳng thức Bunhiacopxki dạng phân thức ta có

$\frac{9}{4a^{2}+b^{2}+c^{2}}= \frac{\left ( a+b+c \right )^{2}}{2a^{2}+\left ( a^{2}+b^{2} \right )+\left ( a^{2}+c^{2} \right )}\leq \frac{a^{2}}{2a^{2}}+\frac{b^{2}}{a^{2}+b^{2}}+\frac{c^{2}}{a^{2}+c^{2}}$

từ đây suy ra

$\sum \frac{9}{4a^{2}+b^{2}+c^{2}}\leq \frac{3}{2}+\sum \left ( \frac{b^{2}}{a^{2}+c^{2}} \right )=\frac{3}{2}+3= \frac{9}{2}$(đpcm)

dấu bằng xảy ra khi $a=b=c=1$



#5
canhhoang30011999

canhhoang30011999

    Thiếu úy

  • Thành viên
  • 634 Bài viết

họ và tên Nguyễn Cảnh Hoàng

lớp 9A trường THCS Đặng Thai Mai thành phố Vinh tỉnh Nghệ An

đề

Cho $a\geq 0$ $b\geq 0$ $c\geq 3$ thỏa mãn $a+b+c= 6$

tìm giá trị lớn nhất của biểu thức $P= abc$

đáp án

$abc= \frac{1}{4}\left ( 2a \right )\left ( 2b \right )c$$\leq \frac{1}{4}\left ( \frac{2a+2b+c}{3} \right )^{3}$

mà $2a+2b+c= 2\left ( a+b+c \right )-c= 12-c\leq 12-3= 9$ nên

$P= abc\leq \frac{1}{4}\left ( \frac{9}{3} \right )^{3}$$= \frac{27}{4}$

dấu băng xảy ra $\Leftrightarrow \left\{\begin{matrix} &2a=2b=c & \\ & c=3 & \\ &a+b+c=6 & \end{matrix}\right.$

$\Leftrightarrow a=b=\frac{3}{2}$,$c= 3$



#6
4869msnssk

4869msnssk

    Bá tước

  • Thành viên
  • 549 Bài viết

Đề bài: Cho a,b,c,p,q dương. CMR $\frac{a}{pb+qc}+\frac{b}{pc+qa}+\frac{c}{pa+qb}\geq \frac{3}{p+q}$

Lời giải:

Ta có:

a=$\sqrt{\frac{a}{pb+qc}}\sqrt{a(pb+qc)}$

b=$\sqrt{\frac{b}{pc+qa}}\sqrt{b(pc+qa)}$

c=$\sqrt{\frac{c}{pa+qb}}\sqrt{c(pa+qb)}$

Áp dụng BĐT Bunhiacopxki cho hai dãy $\sqrt{\frac{a}{pb+qc}};\sqrt{\frac{b}{pc+qa}};\sqrt{\frac{c}{pa+qb}}$ và $\sqrt{a(pb+qc)};\sqrt{b(pc+qa)};\sqrt{c(pa+qb)}$

$\Rightarrow (a+b+c)^{2}=(\sum \sqrt{\frac{a}{pb+qc}})^{2}\leq (\sum \frac{a}{pb+qc})(\sum a(pb+qc))$

$\Rightarrow \frac{(a+b+c)^{2}}{(\sum a(pb+qc))}\leq (\sum \frac{a}{pb+qc})$

$\Rightarrow \frac{(a+b+c)^{2}}{ (p+q)(ab+bc+ca)}\leq (\sum \frac{a}{pb+qc})$

$\Rightarrow \frac{3(ab+bc+ca)}{(p+q)(ab+bc+ca)}\leq \frac{(a+b+c)^{2}}{ (p+q)(ab+bc+ca)}\leq (\sum \frac{a}{pb+qc})$

$\Leftrightarrow \frac{3}{p+q}\leq (\sum \frac{a}{pb+qc})$

dấu "=" xảy ra khi $\left\{\begin{matrix} a=b=c\\ pb+qc=pc+qa=pa+qb \end{matrix}\right.$  hay a=b=c

 

 

Đề thi này không được BTC chấp nhận, lý do: Không rõ tên thật, trường, lớp

 

 


 B.F.H.Stone


#7
neversaynever99

neversaynever99

    Thượng sĩ

  • Thành viên
  • 243 Bài viết

Họ tên: Ngô Lan Anh

Lớp 9A- Trường THCS Đoàn Thị Điểm

Tỉnh:Hưng Yên

Đề bài: Với $a,b,c> 0$, chứng minh rằng:

$\frac{a+3c}{a+b}+\frac{c+3a}{b+c}+\frac{4b}{c+a}\geq 6$

Bài làm

Bất đẳng thức cần chứng minh tương đương với

$\frac{a+3c}{a+b}+2+\frac{c+3a}{b+c}+2+\frac{4b}{c+a}+6\geq 16 \Leftrightarrow A=(3a+2b+3c)(\frac{1}{a+b}+\frac{1}{b+c}+\frac{2}{c+a})\geq 16$

Ta có $P\geqslant (3a+2b+3c)\frac{16}{(a+b)+(b+c)+2(c+a)}= 16$

Đẳng thức xảy ra khi và chỉ khi $a+b= b+c= 2a+2c$

Suy ra $\left\{\begin{matrix} a= c & \\ b+c= 2a+2c \Leftrightarrow b=c+2a=3a & \end{matrix}\right.$



#8
Frankie nole

Frankie nole

    Binh nhất

  • Thành viên
  • 32 Bài viết

họ và tên Nguyễn Thế Huy

đang học lớp 9A trường THCS Đặng Thai Mai thành phố Vinh Nghê An

đề $a+\frac{1}{c\left ( a-b \right )\left ( b-c \right )}$$\geq 4$

với $a> b> c> 0$

đáp án

$VT= a-b+b-c+c+\frac{1}{\left ( a-b \right )\left ( b-c \right )c}$

do $a> b> c> o$ nên áp dụng bđt Cô-si cho 4 số dương ta có

$VT\geq 4$

dấu băng xảy ra $\Leftrightarrow a-b=b-c=c=\frac{1}{\left ( a-b \right )\left ( b-c \right )c}$$\Leftrightarrow a=3,b=2,c=1$



#9
nk0kckungtjnh

nk0kckungtjnh

    Thượng sĩ

  • Thành viên
  • 254 Bài viết

Tên: Trần Minh Nhân Chính
Lớp: 9C

Trường: THCS Tôn Quang Phiệt
Huyện: Thanh Chương- Tỉnh Nghệ An 

Đề bài: 

Cho $x,y\varepsilon R$ thỏa $(x+y)^{3}+4xy\geq 2$ . Tìm giá trị nhỏ nhất của:

                       $ P=3(x^{4}+y^{4}+x^{2}y^{2})-2(x^{2}+y^{2})+1$

Lời giải:               Đặt $K=x^{2}+y^{2}$ . Đặt $M=x+y$

Ta có: 

$P=3[(x^{2}+y^{2})^{2}-x^{2}y^{2}]-2(x^{2}+y^{2})+1$

Áp dụng $BĐT AM-GM$ cho 2 số dương $x^{2},y^{2}$. Ta có: 

$P\geq 3K^{2}-2K+1-0.75K^{2}=2.25K^{2}-2K+1$

$\Leftrightarrow P\geq (K-\frac{1}{2})^{2}+1.25K^{2}+\frac{3}{4}$

Từ giả thiết: Áp dụng $BĐT AM-GM$ cho 2 số dương $x,y$. Ta có: 

$2\leq (x+y)^{3}+4xy\leq (x+y)^{2}(x+y+1)$.

Áp dụng  $BĐT AM-GM$ cho 3 số dương $2M,2M,M+1$. Ta có: 

$2\leq (x+y)^{2}(x+y+1)\leq \frac{1}{4}.\frac{(5M+1)^{3}}{27}$

Từ đó có: $1\leq M\leq \sqrt{2.K}$ Nên $K\geq \frac{1}{4}$

Như vậy: $P\geq \frac{53}{64}$ . $"="$ tại:$x=y=0.5$


             Hãy Đánh Bại Những Gì Yếu Đuối Để Biết Rằng


         Nỗ Lực Hơn Hẳn Tài Năng

- Nhân Chính -

 


#10
nk0kckungtjnh

nk0kckungtjnh

    Thượng sĩ

  • Thành viên
  • 254 Bài viết

Tên: Trần Minh Nhân Chính

Lớp 9C

Trường THCS Tôn Quang Phiệt

Huyện Thanh Chương

Tỉnh Nghệ An

Đề bài:

Cho $x,y$ là các số thực không âm thỏa mãn: 

$x^{2}+y^{2}=5$. Chứng minh rằng:$x^{3}+y^{6}\geq 9$

Lời giải: 

BĐT cần chứng minh tương đương với: 

$\frac{5}{9}(x^{3}+y^{6})\geq (x^{2}+y^{2})$

Xét: $\frac{5}{18}x^{3}+ \frac{5}{18}x^{3}+a^{3}\geq 3\sqrt[3]{\frac{25}{324}.x^{6}.a^{3}}$

$\Leftrightarrow \frac{5}{9}x^{6}\geq 3.\sqrt[3]{\frac{25}{324}}.x^{2}.a-a^{3}$ (1)  

Xét: $\frac{5}{9}y^{6}+b^{3}+b^{3}\geq 3\sqrt[3]{\frac{5}{9}}.b^{2}.y^{2}$             $\Leftrightarrow \frac{5}{9}y^{6}\geq 3\sqrt[3]{\frac{5}{9}}.b^{2}.y^{2}-2b^{3}$  (2)

 

Cộng các vế của (1) và (2). Ta có:

$\frac{5}{9}(y^{6}+x^{3})\geq 3\sqrt[3]{\frac{5}{9}}.b^{2}.y^{2}-2b^{3}+3.\sqrt[3]{\frac{25}{324}}.x^{2}.a-a^{3}$

Nhận thấy rằng nếu:

$\left\{\begin{matrix} \sqrt[3]{\frac{5}{9}}.b^{2}= \sqrt[3]{\frac{25}{324}}.a & \\ a^{3}+2b^{3}=x^{2}+y^{2}=5& \end{matrix}\right.$

Thì bài toán đúng. Nói cách khác, nếu tồn tại a,b thỏa mãn hệ trên thì bài toán được giải quyết.

Ta có: $\frac{5}{9}b^{6}=\frac{25}{324}a^{3}\Leftrightarrow a^{3}=7.2b^{6}$  (3)

Vì vậy: $7.2b^{6}+2b^{3}-5=0$. Giải phương trình bậc 2 một ẩn ( $b^{3}=t$ ). Tìm được $b$. Thay vào (3) tìm được $a$. 

Bài toán được giải quyết. 

 

 


             Hãy Đánh Bại Những Gì Yếu Đuối Để Biết Rằng


         Nỗ Lực Hơn Hẳn Tài Năng

- Nhân Chính -

 


#11
Rias Gremory

Rias Gremory

    Del Name

  • Thành viên
  • 1384 Bài viết

1.Họ và tên : Phạm Văn Hùng

2.Lớp 9C Trường THCS Cẩm Nhượng , Cẩm Xuyên , Hà Tĩnh

3.Đề thi : Cho x,y,z là các số thực dương thõa mãn điều kiện :

$\frac{2}{x}+\frac{1}{y}\leq 1$ và $\frac{4}{z}+y\leq 2$

Tìm GTNN của biểu thức : $P(x,y,z)=x+9y+z$

4.

Do x,y,z dương nên ta có : $1\geq \frac{2}{x}+\frac{1}{y}> \frac{1}{y}\Rightarrow y> 1\Rightarrow y\epsilon (1;2)$

Ta có :$\frac{2}{x}\leq 1-\frac{1}{y}=\frac{y-1}{y}\Rightarrow x\geq \frac{2y}{y-1}=2+\frac{2}{y-1}$

$\frac{4}{z}\leq 2-y\Rightarrow z\geq \frac{4}{2-y}$

Do đó: $P(x,y,z)\geq \frac{2}{y-1}+\frac{4}{2-y}+9y+2$

$= 2(\frac{1}{y-1}+9(y-1))+\frac{4}{2-y}+9(2-y)+2\geq 26$

Dấu = xảy ra $\Leftrightarrow x=\frac{2y}{y-1};z=\frac{4}{2-y};\frac{1}{y-1}=9(y-1)$

và $\frac{4}{2-y}=9(2-y)\Leftrightarrow x=8;y=\frac{4}{3};z=6$

Vậy MinP=26 đạt được khi x=8 ; $y=\frac{4}{3}$ và z=6



#12
pdtienArsFC

pdtienArsFC

    Trung sĩ

  • Thành viên
  • 133 Bài viết

1,  Phan Đức Tiến

2,  Lớp 9C, THCS Bạch Liêu, Yên Thành, Nghệ An.

3,  Đề ra:

Với a, b, c >0. Chứng minh rằng:

$\frac{1}{\sqrt{a}}+\frac{1}{\sqrt{b}}+\frac{1}{\sqrt{c}}\geq \sqrt{3}(\frac{1}{\sqrt{a+2b}}+\frac{1}{\sqrt{b+2c}}+\frac{1}{\sqrt{c+2a}})$

4, Đáp án:

Ta có: $(\sqrt{a}+2\sqrt{b})^{2}\leq (1+2)(a+2b)=3(a+2b)$ 

           $\Leftrightarrow \sqrt{a}+2\sqrt{b}\leq \sqrt{3}\sqrt{a+2b}$    (1)

Mặt khác:

         $(\sqrt{a}+\sqrt{b}+\sqrt{b})(\frac{1}{\sqrt{a}}+\frac{1}{\sqrt{b}}+\frac{1}{\sqrt{b}})\geq 9$

         $\Leftrightarrow \frac{1}{\sqrt{a}}+\frac{2}{\sqrt{b}}\geq \frac{9}{\sqrt{a}+2\sqrt{b}}$     (2)

Từ (1), (2) ta có:

        $\frac{1}{\sqrt{a}}+\frac{2}{\sqrt{b}}\geq \frac{9}{\sqrt{a}+2\sqrt{b}}\geq \frac{9}{\sqrt{3}\sqrt{a+2b}}=\frac{3\sqrt{3}}{\sqrt{a+2b}}$

Chứng minh tương tự, ta có:

       $\frac{1}{\sqrt{b}}+\frac{2}{\sqrt{c}}\geq \frac{9}{\sqrt{b}+2\sqrt{c}}\geq \frac{9}{\sqrt{3}\sqrt{b+2c}}=\frac{3\sqrt{3}}{\sqrt{b+2c}}$

       $\frac{1}{\sqrt{c}}+\frac{2}{\sqrt{a}}\geq \frac{9}{\sqrt{c}+2\sqrt{a}}\geq \frac{9}{\sqrt{3}\sqrt{c+2a}}=\frac{3\sqrt{3}}{\sqrt{c+2a}}$

Cộng vế với vế, ta được điều phải chứng minh.

Dấu "=" xảy ra $\Leftrightarrow a=b=c$


                           80b68e1e79774daab705a98543684359.0.gif

 


#13
hoatuyet1483

hoatuyet1483

    Binh nhất

  • Thành viên
  • 26 Bài viết

1.Họ và tên: Nguyễn Kiều Trinh

2. lớp 9a, THCS Sông Hiến , thành phố Cao Bằng , tỉnh Cao Bằng
3. Đề:

Cho x, y, z >0 thỏa mãn điều kiện  x2 + y2 + z2   $\leq 3$ , tìm giá trị nhỏ nhất của biểu thức "

P= $\frac{1}{1+xy}$$\frac{1}{1+yz}$$\frac{1}{1+zx}$
 

4. Đáp án:

Đặt 1 +xy=a ; 1 +yz =b; 1+ zx=c thì a+b+c= 3+ xy +yz+ zx
Áp dụng bất đẳng thức (a+b+c)($\frac{1}{a}$+$\frac{1}{b}$+$\frac{1}{c}$ ) $\geq 9$

ta được (3+xy+ yz +zx ). P $\geq 9$

Nên P $\geq \frac{9}{3+xy+yz+xz} \geq \frac{9}{3+x^{2}+y^{2}+z^{2}} \geq \frac{3}{2}$

Dấu = xảy ra khi và chỉ khi x =y=z=1
Vậy Min P = $\frac{3}{2}$  <=> x=y=z=1

 

 

\



#14
huukhangvn

huukhangvn

    Trung sĩ

  • Thành viên
  • 103 Bài viết

Họ và tên:Nguyễn Hữu Khang

Lớp 8a1

Trường THCS Trần Văn Ơn-Đắc Lắc

Đề:Cho |x|+|y|+|z|=6 Tìm GTNN : C=|x−1|+|y−1|+|z−1|

giải:

ta có:Giả sử x,y,z âm;lúc này C=9(1)

Giả sử x <0;y,z<0.Khi đó C=5(2)

giả sử x>0,y,z<0.lúc này C=4(3)

giả sử x,z,,y>0;lúc này C=3(4)

từ (1),(2),(3),(4)ta thu được:Min C=3



#15
angleofdarkness

angleofdarkness

    Thượng sĩ

  • Thành viên
  • 246 Bài viết

1. Họ và tên thật: Chu Thanh Huyền.

 

2. Đang học lớp 9a, trường THCS Yên Phong, huyện Yên Phong, tỉnh Bắc Ninh.

 

3. Đề: "Cho x, y, z là các số dương thỏa mãn điều kiện xyz = 1. Tìm giá trị nhỏ nhất của biểu thức:  $E=\frac{1}{x^3(y+z)}+\frac{1}{y^3(z+x)}+\frac{1}{z^3(x+y)}.$ "

 

4. Đáp án:

 

Đặt a = $\frac{1}{x}$; b = $\frac{1}{y}$; c = $\frac{1}{z}$ $\Rightarrow abc=1.$

 

$\Rightarrow$ x + y = c(a + b); y + z = a(b + c); z + x = b(c + a).

 

$\Rightarrow$ $E=\frac{a^2}{b+c}+\frac{b^2}{c+a}+\frac{c^2}{a+b}.$

 

Áp dụng BĐT Nesbit có $\frac{a}{b+c}+\frac{b}{c+a}+\frac{c}{a+b}$ $\geq \frac{3}{2}.$

 

Nhân cả 2 vế với a + b + c > 0 ta được: 

 

$\frac{a(a+b+c)}{b+c}+\frac{b(a+b+c)}{c+a}+\frac{c(a+b+c)}{a+b}$ $\geq \frac{3}{2}.(a+b+c)$

 

$\Rightarrow$ $\frac{a^2}{b+c}+\frac{b^2}{c+a}+\frac{c^2}{a+b}$ $\geq \frac{a+b+c}{2}$ $\geq \frac{\sqrt[3]{abc}}{2}=\frac{3}{2}$

 

Hay có E $\geq \frac{3}{2}.$

 

$\Rightarrow$ min E = $\frac{3}{2}$ khi a = b = c = 1.

 

 

 

 

 

 

 

 

 

 

 

 

 

 

 



#16
anhtukhon1

anhtukhon1

    Sĩ quan

  • Thành viên
  • 480 Bài viết

Cho bàn cờ 8x8

Trong ô cờ thứ nhất đặt 1 hạt ngô

Trong ô cờ thứ hai đặt 2 hạt ngô

Trong ô cờ thứ ba đặt 4 hạt ngô

...

Trong ô cờ thứ 64 đặt 2^63 hạt ngô

Một con mã ô đầu tiên của bàn cờ, nó đi lòng vòng và ăn các hạt ngô trong ô nó nhảy đến( con mã di chuyển theo hình chữ L) nhưng nó không ăn ở ô đầu tiên và không nhảy trở lại ô đầu tiên. Sau mỗi lần nó ăn người ta lại đặt số ngô bằng số ngô ban đầu vào trong ô đó. Sau khi con mã đi xong nó quay trở về ô đầu tiên và ăn nốt hạt ngô ở ô đó.

Hãy CM rằng số ngô mà con mã ăn chia hết cho 3.

Lời giải

Ta tô màu các ô các màu đen trắng xen kẽ nhau.

Số ô trong ô trắng chia 3 dư 1(2^k)

Số ô trong ô đen chia 3 dư -1(2^(2k+1)) 

Con mã nhảy 2 ô khác màu nhau suy ra số ô đen mà nó nhảy bằng số ô trắng mà nó nhảy

Vậy tổng số ngô nó ăn sẽ chia hết cho 3

Họ và tên: Hữu Tường Tú

Trường THCS Tân Lập

Lớp 7A



#17
namcpnh

namcpnh

    Red Devil

  • Hiệp sỹ
  • 1153 Bài viết

Các bài không soạn bằng Latex sẽ không được chọn vào MSS marathon Shortlist 2014.


Cùng chung sức làm chuyên đề hay cho diễn đàn tại :

Dãy số-giới hạn, Đa thức , Hình học , Phương trình hàm , PT-HPT-BPT , Số học.

Wolframalpha đây





0 người đang xem chủ đề

0 thành viên, 0 khách, 0 thành viên ẩn danh